You are on page 1of 11

2 Infinite Series

2.1 Definitions & convergence


Definition 2.1.1. Let {an } be a sequence of real numbers.

a) An expression of the form


a1 + a2 + . . . + an + . . .
is called an infinite series.

b) The number an is called as the nth term of the series.


n
P
c) The sequence {sn }, defined by sn = ak , is called the sequence of partial sums of the series.
k=1

d) If the sequence of partial sums converges to a limit L, we say that the series converges and its
sum is L.

e) If the sequence of partial sums does not converge, we say that the series diverges.

Examples 2.1.2.
∞ 1
xn converges to
P
1) If 0 < x < 1, then .
n=0 1−x
n
xk . Here
P
Solution. Let us consider the sequence of partial sums {sn }, where sn =
k=1

n
X 1 − xn+1 1 xn+1
sn = xk = = − , n ∈ N.
1−x 1−x 1−x
k=1

1
As, 0 < x < 1, xn+1 → 0 as n → ∞. Hence sn → . Thus the given series converges to
1−x
1
. ///
1−x

P∞ 1
2) The series diverges.
n=1 n
n
P 1
Solution. Consider the sequence of partial sums {sn }, where sn = k. Now, let us examine
k=1
the subsequence s2n of {sn }. Here

s2 = 1 + 1/2 = 3/2,
s4 = 1 + 1/2 + 1/3 + 1/4 > 3/2 + 1/4 + 1/4 = 2.

1
Suppose s2n > (n + 2)/2, then
2 n
X 1
s2n+1 = s2n +
2n + k
k=1
2n
n+2 X 1
> +
2 2n+1
k=1
n+2 2n (n + 1) + 2
= + n+1 = .
2 2 2
Thus the subsequence {s2n } is not bounded above and as it is also increasing, it diverges. Hence
P∞ 1
the sequence diverges, i.e., the series diverges. ///
n=1 n


P 1
3) (Telescopic series:) Show that the series converges to 1.
n=1 n(n + 1)

Solution. Consider the sequence of partial sums {sn }. Then


n k  
X 1 X 1 1 1
sn = = − =1− → 1.
k(k + 1) k k+1 n+1
k=1 k=1

Summarizing this observation, one has the following theorem on Telescopic series

Theorem 2.1.3. Suppose {an } is a sequence of non-negative real numbers such that an → L. Then
P
the series (an − an+1 ) converges to a1 − L.

Lemma 2.1.4.

P ∞
P ∞
P
1) If an converges to L and bn converges to M, then the series (an + bn ) converges to
n=1 n=1 n=1
L + M.

P ∞
P
2) If an converges to L and if c ∈ R, then the series can converges to cL.
n=1 n=1

P
Lemma 2.1.5. If an converges, then lim an = 0.
n=1 n→∞


P
Proof. Suppose an = L. Then the sequence of partial sums {sn } also converges to L. Now
n=1

an = sn − sn−1 → L − L = 0. ///

xn diverges.
P
Example 2.1.6. If x > 1, then the series
n=1

xn converges. Then the nth term, i.e., xn → 0.
P
Solution. Assume to the contrary that the series
n=1
But as x > 1, xn ≥ 1 for all n ∈ N and hence lim xn ≥ 1, which is a contradiction. Hence the series
n→∞

xn diverges.
P
///
n=1

2
As a first result we have the following comparison theorem:
P
Theorem 2.1.7. Let {an }, {bn } be sequences of positive reals such that an ≤ bn . If bn converges
P
then an converges.
P P
Proof. Let sn = a1 + a2 + .... + an and tn = b1 + b2 + .... + bn be the partial sum of an , bn
P
respectively. Then sn ≤ tn . Since bn converges, we have {tn } converges and is bounded. Now since
{sn } is monotonically increasing sequence that is bounded above, we get the convergence of {sn } and
P
hence the convergence of an . ///

Theorem 2.1.8. Let {an }∞
P
1 be an decreasing sequence of positive numbers. Then an converges if
n=1

2n a2n converges.
P
and only if
n=0
P P n
Proof. Let sn and tn be the sequence of partial sums of an and 2 a2n respectively. Then sn and
tn are monotonically increasing sequences. We know that such sequences converge if they are bounded
from above. proof follows from the observation that
2 n
X
s2n = an = a1 + a2 + (a3 + a4 ) + (a5 + a6 + a7 + a8 ) + .... + (a2n−1 +1 + .... + a2n )
k=1
≥ a1 + a2 + 2a4 + 4a8 + 8a16 + ....2n−1 a2n
1
= a1 + tn . (2.1)
2
Therefore, if {sn } converges then {s2n } converges and hence bounded from above. Now convergence
of {tn } follows from 2.1, {tn }.
On the other hand,

s2n −1 = a1 + (a2 + a3 ) + (a4 + a5 + a6 + a7 ) + (a8 + ....a15 ) + (a2n−1 + .... + a2n −1 )


≤ a1 + 2a2 + 4a4 + 8a8 + .... + 2n−1 a2n−1 = a1 + tn−1

So if {tn } converges, then {s2n −1 } converges. Now the conclusion follows from sn ≤ s2n+1 −1 and the
fact that {sn } is monotonically increasing sequence. ///

Examples 2.1.9.
∞ 1 ∞ ∞
n 1 1
P P P
1) Consider the series p
, p > 0. Then, we have 2 n )p
= n )p−1
which converges for
n=1 n n=1 (2 n=1 (2
p > 1 and diverges for p ≤ 1.
∞1 ∞ 1 1 P ∞ 1
2n n
P P
2) Consider the series . Here = which diverges. Hence the
n=2 n log n n=2 2 log 2n log 2 n=2 n
given series diverges.

3
2.2 Absolute convergence

P ∞
P ∞
P
Definition 2.2.1. a) Let an be a series of real numbers. If |an | converges, we say that an
n=1 n=1 n=1
converges absolutely.

P ∞
P ∞
P
b) If an converges but |an | diverges, we say that an converges conditionally.
n=1 n=1 n=1
Examples 2.2.2.
P∞ (−1)n
1) The series converges absolutely.
n=1 n!
P∞ (−1)n
2) The series n=1 n2
converges absolutely.

P ∞
P
Theorem 2.2.3. If an converges absolutely, then an converges.
n=1 n=1
n
|ak |. As the series converges absolutely, the sequence {tn }∞
P
Proof. Let tn = 1 is Cauchy. Thus, given
k=1
 > 0, there exists N ∈ N such that

|tm − tn | <  ∀ m, n ≥ N.

Let m > n. Then m


X m
X
|sm − sn | = ai ≤ |ai | = |tm − tn | < .


i=n+1 i=n+1

Thus the sequence {sn }∞
P
1 is Cauchy and hence converges. Thus an converges. ///
1

P
Theorem 2.2.4. Let an be a series of real numbers. Let pn = max{an , 0} and qn = min{an , 0}.
1
P P P
a) If an converges absolutely, then both pn and qn converges.
P P P
b) If an diverges then one of the pn or qn diverges.
P P P
b) If an converges conditionally then both pn and qn diverges.

Proof.

a) Observe that pn = (an + |an |)/2 and qn = (an − |an |)/2. Thus the convergence of the two series
follows from the hypothesis.

b) Proof is easy.

c) We leave to this as an exercise.

///

4
Tests for absolute convergence

P P
Theorem 2.2.5 (Comparison test). Let an be a series of real numbers. Then, an converges
P
absolutely if there is an absolutely convergent series cn with |an | ≤ |cn | for all n ≥ N, N ∈ N.
Proof follows as in Theorem 2.1.7
Examples 2.2.6.

P 7 7 1 1 P1
1) The series diverges because = ≥ for all n ∈ N and diverges.
n=1 7n − 2 7n − 2 n − 2/7 n n
P∞ 1 1 1 P∞ 1
2) The series converges because ≤ n and n
converges.
n=0 n! n! 2 n=0 2

Theorem 2.2.7 (Limit comparison test). Let {an } and {bn } be two sequences of positive numbers.
Then
an P P
a) if lim = c > 0, an and bn both converge or diverge together;
n→∞ bn

an P P
b) if lim = 0 and bn converges, then an converges.
n→∞ bn

an P P
c) if lim = ∞ and bn diverges, then an diverges.
n→∞ bn

an
Proof. (a) As lim = c > 0, for  = 2c > 0, there exists N ∈ N such that
n→∞ bn

an c
n ≥ N =⇒ − c < .

bn 2

Thus, for n ≥ N,
−c an c
≤ −c≤
2 bn 2
or equivalently
cbn 3cbn
≤ an ≤ .
2 2
Hence the conclusion follows from the comparison test.
an
b) Given that lim = 0. Hence for  = 21 , there exists N ∈ N such that
n→∞ bn

an 1
n ≥ N =⇒ <
bn 2
or equivalently,
bn
n ≥ N =⇒ an ≤ .
2
Thus the desired conclusion follows from the comparison test.

5
an
c) Here we are given that lim = ∞. Hence for any real number M > 0, there exists N ∈ N
n→∞ bn
such that
an
n ≥ N =⇒ ≥M
bn
or equivalently,
n ≥ N =⇒ an ≥ M bn .
P P
Thus if |bn | diverges, then |an | diverges by comparison test. ///

Examples 2.2.8.
 
2n + 1
P∞ 2n + 1 2n + 1 1 an (n + 1)2
1) Consider the series . Here an = . Let bn = . Then = =
n=1 (n + 1)
2 (n + 1) 2 n bn 1
n
2n2 + n P1
→ 2 as n → ∞. Further, n diverges. Thus by limit comparison theorem, the
n2 + 2n + 1
given series diverges.

P 1 1 1 an 2n
2) Consider the series . Here a n = . Let bn = . Then = → 1. Further,
1 2n − 1 2n − 1 2n bn 2n − 1
P 1
converges and hence the given series converges.
2n
P e−n −n
3) Consider the series n2
. Here an = en2 and bn = n12 . Then abnn = e−n → 0 as n → ∞.
P 1
Further, n2
converges and hence the given series converges.
P e−n e−n 1 an −n → 0 as n → ∞.
4) Consider the series n . Here an = n and bn = n2 . Then bn = ne
P 1
Further, n2
converges and hence the given series converges.

P
Theorem 2.2.9 (Ratio test). Let an be a series of real numbers. Let
1

an+1 an+1
a = lim inf
and A = lim sup
.
n→∞ an n→∞ an
Then

P
a) an converges absolutely if A < 1;
1

P
b) |an | diverges if a > 1;
1

c) the test fails in all other cases.

Proof. a) If A < 1, choose B such


thata A < B < 1. Then there exists an  > 0 such that B = A + 
an+1
and also N ∈ N such that an ≤ B for all n ≥ N. Further, for any k ∈ N,

k k
aN +k Y aN +i Y

aN
= ≤
aN +i−1 B = Bk.
i=1 i=1

6

Thus |aN +k | ≤ B k |aN |, k ∈ N. But |aN |B k < ∞ as B < 1. Thus by comparison test, the series
P
k=0

P
an converges.
1
b) If a > 1, choose b such that 1 < b < a. There exits N ∈ N such that an+1
an ≥ b for all n ≥ N.

Further, for any k ∈ N,
k k
aN +k Y aN +i Y

aN
= ≥
aN +i−1 b = bk .
i=1 i=1

aN bk diverges. Thus, again, by the comparison test,
P
Thus |aN +k | ≥ |aN |, k ∈ N. But, as b > 1,
k=0

P
the series an diverges.
1
P1 an+1 P1
c) Case1: a = A = 1 Consider the series . Here lim = 1. But diverges. For the
n n→∞ an n
P 1 an+1
series , which converges, again lim = 1.
n2 n→∞P ann
Case 2: A > 1 If we consider the series 2 then A = 2 > 1 and the series diverges. If we take
2 1 1 1
s=1+2+ + 2( )2 + ( )3 + 2( )3 + ....
5 5 5 5
Then it is easy to see that the series converges as
1 1 1 1 1
s = 1 + ( ) + ( )3 + ..... + 2 + 2( ) + 2( )2 + 2( )3 + ...
5 5 5 5 5
But A = 2. Similarly one can construct examples when a < 1. ///

Examples 2.2.10.
∞ nn
P
a) Consider the series . Here
1 n!
n
(n + 1)n+1 n! 1 n
  
an+1 n+1
= = = 1+ → e,
an (n + 1)! nn n n

which is greater than 1. So a = A = e > 1. Thus the given series diverges.


∞ xn
P
b) Consider the series , x ∈ R. Here
0 n!

an+1 xn+1 n! x
= = → 0.
an (n + 1)! xn n+1

Therefore a = A = 0 < 1. Thus, for all x ∈ R, the given series converges.



P p
Theorem 2.2.11 (Root test). Let an be a series of real numbers. Let A = lim sup n |an |. Then
1 n→∞

a) the series converges absolutely if A < 1;

7
b) the series diverges if A > 1;

c) the test fails if A = 1.


p
Proof. a) If A < 1, choose B such that A < B < 1. Then there exists N ∈ N such that n |an | < B for
all n ≥ N. This implies |an | < B n for all n ≥ N. As B < 1, the series converges by comparison test.
p
b) If A > 1, there exists infinitely many n ∈ N such that n |an | > 1. But this implies that |an | > 1
P
for infinitely many values of n and hence aN 9 0, i.e., an diverges.
P1
c) Consider the series . Here A = 1 and the series diverges. On the other hand, for the series
n
P 1
, again A = 1, but the series converges. ///
n2
Examples 2.2.12.
s
∞ xn xn xn

= √x → |x|. Thus the
P n

1) Consider the series , x ∈ R. Here an = . Therefore, n n
1 n n n
series converges for |x| < 1 and diverges for |x| > 1.
P∞ xn xn p x
n
2) Consider the series , x ∈ Here a = . Then, |a | = → 0. Thus the series

n
R. n n
1 n nn n
converges for any x ∈ R.
 n
 n is odd √
3) Consider the series
P
an , where an = 4n . Then lim sup n an = 21 . Therefore the
1
 n is even n→∞
2n
series converges.
P −n−(−1)n 1
4) The series 3 . Then it is not difficult to see that lim sup |an | n = 1/3. However ratio
test fails in this case.

Remark 2.1. We note that the root test is stronger than the ratio test. for example, take the series
P
an where
(
2−n n odd
an =
2−n+2 n even
Then it is easy to see that
|an+1 |
lim sup = 2, but lim sup |an |1/n = 1/2.
|an |
So the root test implies that the series converges but ratio test is inconclusive.
Alternating series:
Definition 2.2.13. An alternating series is an infinite series whose terms alternate in sign.
Theorem 2.2.14. Suppose {an } is a sequence of positive numbers such that

(a) an ≥ an+1 for all n ∈ N and

(b) lim an = 0,
n→∞

8

(−1)n+1 an converges.
P
then the alternating series
n=1

Proof. Consider the partial sums with odd index, s1 , s3 , s5 , . . . . Now, for any n ∈ N,

s2n+1 = s2n−1 − a2n + a2n+1 ≤ s2n−1 (by (a)).

Thus the sequence {s2n−1 }∞


1 forms a non-increasing sequence. Also, notice that

n−1
X
s2n−1 = (a2i−1 − a2i ) + a2n−1 .
i=1

Since each quantity in the parenthesis is non-negative and a2n−1 > 0, the sequence {s2n−1 } is bounded
below by 0. Hence {s2n−1 }∞1 is convergent.
Now, consider the partial sums with even index, s2 , s4 , s6 , . . . . For any n ∈ N,

s2n+2 = s2n + a2n+1 − a2n+2 ≥ s2n (by (a)).

Thus the sequence {s2n }∞


1 forms a non-decreasing sequence. Further,

n−1
X
s2n = a1 − (a2i − a2i+1 ) − a2n ≤ a1 ,
i=1

which means that s2n is bounded above by a1 . Therefore, {s2n } is convergent.


Let L = lim s2n and M = lim S2n−1 . By ((b)),

0 = lim a2n = lim (s2n − s2n−1 ) = L − M.



(−1)n+1 an converges.
P
Thus L = M and hence the alternating series ///
n=1

Examples 2.2.15.

(−1)n+1 21/n . Here an = 21/n → 1 as n → ∞. Hence the above theorem
P
1) Consider the series
n=1
does not apply. Anyhow, one can show that the series diverges.
∞ (−1)n+1
. The a0n s of this series satisfies the hypothesis of the above
P
2) Consider the series
n=1 n
theorem and hence the series converges.

Examples 2.2.16.
P∞ (−1)n+1
1) The series converges conditionally.
n=1 n
P∞ (−1)2n−1
2) The series converges conditionally.
n=1 2n − 1

The following is a more genreal test than the previous theorem.

9
Theorem 2.2.17. (Dirichlet test)
Let {an } and {bn } be sequences of real numbers such that
n
X
1. the sequence sn = ak is bounded,
k=1

2. the sequence bn is decreasing and bn → 0.


P
Then the series an bn converges.

Proof. Since An is bounded, there exists M > 0, such that |An | ≤ M for all n. Now note that

a1 b1 + a2 b2 + ... + an bn = s1 (b1 − b2 ) + s2 (b2 − b3 ) + ....sn−1 (bn−1 − bn ) + sn bn

Since bn is decreasing, bn − bn+1 ≥ 0. Therefore

|a1 b1 + a2 b2 + ... + an bn | ≤ M b1

Since bn → 0, for any  > 0, we get N such that |bn | ≤  for all n ≥ N . Now we can easily see,
m
X
| ak bk | = M |bn | ≤ M 
n
P
Therefore, by Cauchy’s test, the series an bn converges. ///

Examples 2.2.18.
X cos nπ
1
1) Consider the series . Here take an = cos nπ and bn = log n . Then
log n
n
X
|An | ≤ | cos nπ| ≤ 1
k=1

(check the first 4 terms and then use periodicity of cos x)


and bn decreases to 0. Hence the series conveges. In this case we can see that the series does
not converge absolutely (apply Cauchy’s test).
P 22n n2 1 1 2n n2
2) and an = 2en n!
en n! (log n)2 . Take bn = (log n)2 P . Then bn decreases to 0. To show the
P
boundedness of the partial sums of an , we can apply Ratio test to see that the series an
converges. Hence the sequence of partial sums converge and so will be bounded. Therefore by
P
Dirichlet test the series an bn converges.

Theorem 2.2.19. (Integral Test). If f (x) is decreasing and non-negative on [1, ∞), Then
Z ∞ ∞
X
f (x)dx < ∞ ⇐⇒ f (n) converges.
1 n=1

Details of this theorem will be done after convergence of improper integral.

10
2.3 Problems

P
1. If the terms of the convergent series an are positive and forms a non-increasing sequence,
n=1
then prove that lim 2n a2n = 0.
n→∞

an xn converges.
P
2. If 0 ≤ an ≤ 1 (n ≥ 0) and if 0 ≤ x ≤ 1, then prove that
n=1
∞ ∞
P 1 P 1
3. Test the convergence of the series (1) n(log n)p (2) (log n)x x ∈ R.
n=2 n=2

4. Determine which of the following series diverges


∞ ∞ ∞ √ ∞ ∞ ∞ √
X log n X (log n)2 X n X 1−n X 1 X n
n
(a) (b) (c) 2
(d) n
(e) √ (f ) .
n3/2 n3/2 n +1 n2 n
n n n2
n=1 n=1 n=1 n=1 n=1 n=1

5. Determine which of the following series converges



∞ ∞ ∞  ∞ ∞
2 n−2 n (log n)n

X n X n! X X X n!
(a) (b) (c) (d) (e)
2n 10n n n n (2n + 1)!
n=1 n=1 n=1 n=1 n=1

6. Test the convergence of the infinite series


∞ π ∞ ∞  
X X 1 X1 1 X 1 1
(1) sin n (2) (3) sin √ (4) √ sin( )
2 n ln(n3 ) n n n n
n=0 n=2 n=1

7. Test the convergence of the infinite series


X n2 n! X sin nπ X n(n!)
2
(1) (2) (c)
(n3 + 1)nn n (n2 + 1)[(2n + 1)!]

11

You might also like